Difference between revisions of "2018 AMC 10B Problems/Problem 21"

(Solution 5)
(Solution 2: I will rewrite a similar solution using detailed casework. CREDITS WILL BE RETAINED TO TDENG.)
Line 11: Line 11:
  
 
-Edited by Mathandski
 
-Edited by Mathandski
 
==Solution 2==
 
Let the next largest divisor be <math>k</math>. Suppose <math>\gcd(k,323)=1</math>. Then, as <math>323|n, k|n</math>, therefore, <math>323\cdot k|n.</math> However, because <math>k>323</math>, <math>323k>323\cdot 324>9999</math>. Therefore, <math>\gcd(k,323)>1</math>. Note that <math>323=17\cdot 19</math>. Therefore, the smallest the GCD can be is <math>17</math> and our answer is <math>323+17=\boxed{\text{(C) }340}</math>.
 
  
 
==Solution 3==
 
==Solution 3==

Revision as of 00:37, 18 October 2021

The following problem is from both the 2018 AMC 12B #19 and 2018 AMC 10B #21, so both problems redirect to this page.

Problem

Mary chose an even $4$-digit number $n$. She wrote down all the divisors of $n$ in increasing order from left to right: $1,2,...,\dfrac{n}{2},n$. At some moment Mary wrote $323$ as a divisor of $n$. What is the smallest possible value of the next divisor written to the right of $323$?

$\textbf{(A) } 324 \qquad \textbf{(B) } 330 \qquad \textbf{(C) } 340 \qquad \textbf{(D) } 361 \qquad \textbf{(E) } 646$

Solution 1

Since prime factorizing $323$ gives you $17 \cdot 19$, the desired answer needs to be a multiple of $17$ or $19$, this is because if it is not a multiple of $17$ or $19$, $n$ will be more than a $4$ digit number. For example, if the answer were to instead be $324$, $n$ would have to be a multiple of $2^2 * 3^4 * 17 * 19$ for both $323$ and $324$ to be a valid factor, meaning $n$ would have to be at least $104652$, which is too big. Looking at the answer choices, $\text{(A) }324$ and $\text{(B) }330$ are both not a multiple of neither 17 nor 19, $\text{(C) }340$ is divisible by $17$. $\text{(D) }361$ is divisible by $19$, and $\text{(E) }646$ is divisible by both $17$ and $19$. Since $\boxed{\text{(C) }340}$ is the smallest number divisible by either $17$ or $19$ it is the answer. Checking, we can see that $n$ would be $6460$, a four-digit number. Note that $n$ is also divisible by $2$, one of the listed divisors of $n$. (If $n$ was not divisible by $2$, we would need to look for a different divisor)

-Edited by Mathandski

Solution 3

Again, recognize $323=17 \cdot 19$. The 4-digit number is even, so its prime factorization must then be $17 \cdot 19 \cdot 2 \cdot n$. Also, $1000\leq 646n \leq 9998$, so $2 \leq n \leq 15$. Since $15 \cdot 2=30$, the prime factorization of the number after $323$ needs to have either $17$ or $19$. The next highest product after $17 \cdot 19$ is $17 \cdot 2  \cdot 10 =340$ or $19 \cdot 2  \cdot 9 =342$ $\implies \boxed{\text{(C) }340}$.


You can also tell by inspection that $19\cdot18 > 20\cdot17$, because $19\cdot18$ is closer to the side lengths of a square, which maximizes the product.

~bjhhar

Solution 4 (Using the answer choices)

Note that $323$ multiplied by any of the answer choices results in a 5 or 6 digit $n$. So, we need a choice that shares a factor(s) with 323, such that the factors we'll need to add to the prime factorization of $n$(in result to adding the chosen divisor) won't cause our number to multiply to more than 4 digits. The prime factorization of $323$ is $17\cdot19$, and since we know $n$ is even, our answer needs to be (1) even & (2) with a factor of $17$ or $19$. We see 340 achieves this and is the smallest to do so (646 being the other). So, we get $\boxed{\text{(C) }340}$ -- OGBooger -- minor changes by Pearl2008

Solution 5

We see that $323$ is $18^2 -1$, which means it's prime factorization is $(18-1)(18+1)$ which is $17 \cdot 19.$ The factors that are possible is an even number with $17$ as a factor or $19$ as a factor. The smallest factors larger than $17 \cdot 19$ are $17 \cdot 20$ or $19 \cdot 18$, and we can see that $17 \cdot 20$ is smaller than $19 \cdot 18$ since $19 \cdot 18$ is closer to a square, so therefore our answer is $\boxed{\text{(C) }340}$.

Video Solution 1

https://www.youtube.com/watch?v=qlHE_sAXiY8

Video Solution 2

https://www.youtube.com/watch?v=KHaLXNAkDWE

Video Solution 3

https://www.youtube.com/watch?v=vc1FHO9YYKQ

~bunny1

See Also

2018 AMC 10B (ProblemsAnswer KeyResources)
Preceded by
Problem 20
Followed by
Problem 22
1 2 3 4 5 6 7 8 9 10 11 12 13 14 15 16 17 18 19 20 21 22 23 24 25
All AMC 10 Problems and Solutions
2018 AMC 12B (ProblemsAnswer KeyResources)
Preceded by
Problem 18
Followed by
Problem 20
1 2 3 4 5 6 7 8 9 10 11 12 13 14 15 16 17 18 19 20 21 22 23 24 25
All AMC 12 Problems and Solutions

The problems on this page are copyrighted by the Mathematical Association of America's American Mathematics Competitions. AMC logo.png